A group of friends wants to go to the amusement park. They have no more than $160
to spend on parking and admission. Parking is $8, and tickets cost $38 per person,
including tax. Write and solve an inequality which can be used to determine p, the
number of people who can go to the amusement park.

Answers

Answer 1

Answer:

I tried figuring out this problem but it is a tad confusing. sorry :/

Step-by-step explanation:


Related Questions

Guys please help i dont understand im bad at math

Answers

The first on is B and the second one is C
first one is b and the other is d

The question I’m trying to figure out is What is y?

Answers

Answer:

y = 136

Step-by-step explanation:

x + 54 + 82 = 180

x + 136 = 180

x = 44

x + y = 180

44 + y = 180

y = 180 - 44

y = 136

The dimensions of a rectangular prism are 5 cm, 5 cm, and 2 cm.
Determine the surface area.
OA.
A. 50 cm
B. 25 cm
OC. 60 cm
D. 120 cm
O E. 90 cm

Answers

Answer:

E

Step-by-step explanation:

Givens

L = 5 cm

W = 5 cm

H = 2 cm

Formula

SA = 2L*W + 2L*H + 2*W*H

Solution

SA = 2*5*5 + 2*5*2 + 2*5*2

SA = 50 + 20 + 20

SA = 90 cm^3

(factoring polinonomials) help help help help help help please:(((​

Answers

I think it’s b sorry if I’m wrong

Find the (a) mean, (b) variance, and (c) standard deviation of the binomial distribution for the given random variable, and (d) interpret the results. Sixty-three percent of U.S. mothers with school-age children choose fast food as a dining option for their families one to three times a week. You randomly select five U.S. mothers with school-age children and ask whether they choose fast food as a dining option for their families one to three times a week. The random variable represents the number of U.S. mothers who choose fast food as a dining option for their families one to three times a week.

Answers

Answer:

a) The mean is 3.15.

b) The variance is 1.1655.

c) The standard deviation is 1.08.

d) The expected number of families in the sample who choose fast food as a dining option for their families one to three times a week is 3.15. The variance of 1.1655 and the standard deviation of 1.08 are the averages from which the sample results should diverge from the mean.

Step-by-step explanation:

Binomial probability distribution

Probability of exactly x sucesses on n repeated trials, with p probability.

The expected value of the binomial distribution is:

[tex]E(X) = np[/tex]

The variance of the binomial distribution is:

[tex]V(X) = np(1-p)[/tex]

The standard deviation of the binomial distribution is:

[tex]\sqrt{V(X)} = \sqrt{np(1-p)}[/tex]

Sixty-three percent of U.S. mothers with school-age children choose fast food as a dining option for their families one to three times a week.

This means that [tex]p = 0.63[/tex]

You randomly select five U.S. mothers with school-age children and ask whether they choose fast food as a dining option for their families one to three times a week.

This means that [tex]n = 5[/tex]

(a) mean

[tex]E(X) = np = 5*0.63 = 3.15[/tex]

The mean is 3.15.

(b) variance

[tex]V(X) = np(1-p) = 5*0.63*0.37 = 1.1655[/tex]

The variance is 1.1655.

(c) standard deviation

[tex]\sqrt{V(X)} = \sqrt{np(1-p)} = \sqrt{5*0.63*0.37} = 1.08[/tex]

The standard deviation is 1.08.

(d) interpret the results.

The expected number of families in the sample who choose fast food as a dining option for their families one to three times a week is 3.15. The variance of 1.1655 and the standard deviation of 1.08 are the averages from which the sample results should diverge from the mean.

solve for bc !.??.?.?.?.

Answers

Answer:

A) 12

Step-by-step explanation:

all triangle CBF is similar to triangle GEF by aa similarity.

EF/BF = EG/BC

6/18 = 4/BC

therefore BC must be 12.

Ryan exponential function of the form f(t)= a×b^t to represent the following situation. assume t is in years. an account has an initial investment of $91,000, increasing by 3.76% per year. F(T)=_________​

Answers

Answer

[tex]f(t) = 91000 *1.0376^t[/tex]

Explanation

Given

[tex]a = 91000[/tex] -- initial

[tex]r = 3.76\%[/tex]

Required

The exponential equation

The exponential equation is:

[tex]y =ab^t[/tex]

Where

[tex]b=1 +r[/tex]

We used minus, because the rate increases..

So, we have:

[tex]b=1 +3.76\%[/tex]

[tex]b=1 +0.0376[/tex]

[tex]b=1.0376[/tex]

So:

[tex]y =ab^t[/tex]

[tex]y = 91000 *1.0376^t[/tex]

Hence:

[tex]f(t) = 91000 *1.0376^t[/tex]

Find the diameter of the circle with the given circumference. Use 3.14 for π(PI). C=22 cm

Answers

the answer is 7.006
Answer: 14.01274
Explanation: The formula of the diameter of a circle is 2xradius. We only have the Circumference so we divide 22 by pi which is 7.00637. After that we multiply the radius (7.00637) by 2 which gives us 14.01274. If you round it is equal to 14.0

I need help if right I will mark you as brainlest

Answers

Answer:

More than 275 dollars of here budget goes to emergencies

Step-by-step explanation:

11% of 2750 is $302.50

Solve for x to the nearest degree.

Answers

Answer:

71

Step-by-step explanation:

The square represents 90 degrees.

A triangle’s three angles ALWAYS adds up 180.

To find the missing angle:

180 - 90 = 90

90 - 19 = 71.

Therefore, the answer is 71

If the principal is $200 and the interest rate is 4 percent, what is the simple interest earned in one year?
А.$8
B.$50
C.$800

Answers

Answer:

A

Step-by-step explanation:

Can someone please help me! i’m confused!

Answers

Answer:

i am so sorry about it

its difficult

Find the area (in square inches) of an equiangular triangle with a radius of length 6 in.

Answers

The area of equiangular triangle with radius 6 inches is 27√3 square inches.

What is equiangular triangle?

A triangle with three equal interior angles is called equiangular triangle.

In an equiangular triangle, the measure of each of its interior angle is 60 degrees. Since, all the angles are equal therefore all the side will also be equal.

What is the formula for finding the area of equiangular triangle?

The formula for finding the area of equiangular triangle is

[tex]Area = \frac{\sqrt{3} }{4} a^{2}[/tex]

Where,

a is the side length of the triangle.

Let the side length of the equiangular triangle be a.

According to the given question.

We have a equiangular triangle with a radius 6.

Since, the radius of the equilateral triangle or equiangular triangle is given by

Radius = [tex]\frac{a}{\sqrt{3} }[/tex]

⇒ a = 6√3 inches.

Therefore, the area of equiangualr triangle is given by

[tex]Area = \frac{\sqrt{3} }{4} (6\sqrt{3} )^{2}[/tex]

⇒ [tex]Area = \frac{\sqrt{3} }{4} \times 36 \times 3[/tex]

[tex]\implies Area = 27\sqrt{3}[/tex]  square inches

Hence, the area of equiangular triangle with radius 6 inches is 27√3 square inches.

Find out more information about area of equaingular triangle here:

https://brainly.com/question/2272529

#SPJ2

What is the volume of the right rectangular prism, in cubic centimeters? A prism has a length of 4 centimeters, width of 9 centimeters, and a height of 10 centimeters

Answers

Answer:

360

Step-by-step explanation:

length times width of the base then multiply by the height of the prism.

(4)(9)10

Xander needs to collect at least 120 cans for a food drive to earn community service credit. He has already collected 64 items.

Part A
Choose the inequality and solution to represent the number of cans, x, that Xander must still collect.
a x + 64 ≤ 120; x ≤ 56
b x + 120 ≤ 64; x ≤ –56
c x + 64 ≥ 120; x ≥ 56
d x + 120 ≥ 64; x ≥ –56

Answers

Answer:

b

Step-by-step explanation:

Given the system which is true? Thanks in advance!

Answers

Answer: the system has no solution

Step-by-step explanation:

The slope and the y-intercept is different

rewrite the function by completing the square. f(x)=x^2-9x+14​

Answers

Answer:

f(x)=(x-9/2)²-25/4

Step-by-step explanation:

f(x)=x²-9x+14

f(x)=x²-9x+(-9/2)²-(-9/2)²+14

f(x)=(x-9/2)²-81/4+14

f(x)=(x-9/2)²+(-81+56)/4

f(x)=(x-9/2)²-25/4

Answer:

Step-by-step explanation:

the box and whisker plot below shows the volunteer service hours performed by students at indian trail middle school last summer

Answers

Answer:

circuit compare

with the current in a series circuit, if both circuits have the same

type of cell and the same number of identical flashlight bulbs?

circuit compare

with the current in a series circuit, if both circuits have the same

type of cell and the same number of identical flashlight bulbs?

circuit compare

with the current in a series circuit, if both circuits have the same

type of cell and the same number of identical flashlight bulbs?

circuit compare

with the current in a series circuit, if both circuits have the same

type of cell and the same number of identical flashlight bulbs?

circuit compare

with the current in a series circuit, if both circuits have the same

type of cell and the same number of identical flashlight bulbs?

circuit compare

with the current in a series circuit, if both circuits have the same

type of cell and the same number of identical flashlight bulbs?

circuit compare

with the current in a series circuit, if both circuits have the same

type of cell and the same number of identical flashlight bulbs?

What is the area of this figure?

Select from the drop-down menu to correctly complete the statement.

The area of the figure is

What is the area of this figure?

Select from the drop-down menu to correctly complete the statement.

The area of the figure is Choose...128136153258 in².

What is the area of this figure?

Drag and drop the appropriate number into the box.

A = 

What is the area of this figure?

Drag and drop the appropriate number into the box.

What is the area of this figure?

Drag and drop the appropriate number into the box.

What is the area of this figure?

Select from the drop-down menu to correctly complete the statement.

The area of the figure is

What is the area of this figure?

Select from the drop-down menu to correctly complete the statement.

The area of the figure is

What is the area of this figure?

Select from the drop-down menu to correctly complete the statement.

The area of the figure is Choose...128136153258 in².

What is the area of this figure?

Select from the drop-down menu to correctly complete the statement.

The area of the figure is Choose...128136153258 in².

What is the area of this figure?

Drag and drop the appropriate number into the box.

A = 

What is the area of this figure?

Drag and drop the appropriate number into the box.

What is the area of this figure?

Drag and drop the appropriate number into the box.

What is the area of this figure?

Select from the drop-down menu to correctly complete the statement.

The area of the figure is

What is the area of this figure?

Select from the drop-down menu to correctly complete the statement.

The area of the figure is Choose...128136153258 in².

What is the area of this figure?

Drag and drop the appropriate number into the box.

Rectangle VWXY is dilated by a scale factor of 2/3 to form rectangle V'W'X'Y’. Side VW
measures 21. What is the measure of side V'W'?

Answers

Answer:

y

Step-by-step explanation:

Answer:

V'W' = 14

Steps:

V'W' = VW×2/3

V'W' = 21×2/3

V'W' = 42/3

V'W' = 14

A rocking horse has a weight limit of 60 pounds. What weight is 95% of the limit? *​

Answers

Answer:

57

Step-by-step explanation:

60*0.95 = 57

Answer:

57 pounds is 95 percent of the limit.

Step-by-step explanation:

Solve the system of equations.
2x + 3y = 13
-6x + 4y = 0
A. (2,3)
B.1/2,4
C. (5, 1)
D. (-2, -3)

Answers

Answer:

Step-by-step explanation:

The answer is A for both

Answer:

multiply equation 1by -3 and multiply equation 2 by 1

-3×2x+3y×-3=13×-3

-6x+4y=0

-6x-9y=-39

-6x+4y=0

Step-by-step explanation:

-13y=-39

divide both side by -13

-13y/-13=-39/-13

y=3

put answer of y in equation 1

2x+3y=13

2x+3×3=13

2x+9=13

take like terms of the equation

2x=13-9

2x=4

divide both side by 2

2x/2=4\2

x=2

(x,y)=(2,3)

3/14 x 2/12 multiply then simplify

Answers

Answer:

1/28

Step-by-step explanation:

6/168=1/28

Answer:

hi

Step-by-step explanation:

I think so...

[tex] \frac{3}{14} \times \frac{2}{12} = \frac{1}{7} \times \frac{1}{4} = \frac{1}{28} [/tex]

hope it helps

have a nice day

Me.Bernstein will cut 8 pies into pieces that are each 1/6 of the whole After he cuts the 8 pies. how many pieces will Mr.Bernstein have

Answers

9514 1404 393

Answer:

  48

Step-by-step explanation:

If each piece is 1/6 of the whole pie, then the whole pie has been divided into 6 equal pieces. The number of pieces resulting from 8 pies being cut that way is ...

  (8 pies)(6 pieces/pie) = 8×6 pieces = 48 pieces

Please help. This makes 0 sence also fast because this is a big test I need to pass don’t just guess please?!

Answers

It's B : 5/9 = 0.555… .

To see why, let x = 0.555… . Then 10x = 5.555…, and

10x - x = 5.555… - 0.555…

9x = 5

x = 5/9

Why the other answers are wrong:

• It's not A because 3/10 = 0.3, while 1/3 = 0.333…

• It's not C because 9/100 = 0.09, while 1/11 = 0.090909… :

x = 0.090909…

100x = 9.090909…

100x - x = 9.090909… - 0.090909…

99x = 9

x = 9/99 = 1/11

• It's not D because 47/99 ≠ 0.475475475… :

x = 0.475475475…

1000x = 475.475475475…

1000x - x = 475.475475475… - 0.475475475…

999x = 475

x = 475/999 ≠ 47/99

2) Find AC
C
12° 15°
70
B
А

Answers

The answer is = 12 15


Please help meeeeeeeeeeeeeeeee

Answers

Answer:

Step-by-step explanation:

h = 4.2 in

Volume = 3.6  cubic inches

[tex]\frac{1}{3}\pi r^{2}h = 3.6\\\\\frac{1}{3}*3.14*r^{2}*4.2 = 3.6\\\\r^{2}= \frac{3.6*3}{3.14*4.2}\\\\ = 0.81\\r=\sqrt{0.81}\\\\[/tex]

r = 0.9 in

Answer:

.... ..................

In this combined function, the domain is all real numbers for x except?( x-2) / (x+5)

-2
2
-5
5

Answers

-5 because the denominator can’t equal zero


Johnny Appleseed was creating a fractal tree design:


The first tree, (a1), only has 1 leaf (the green point). The second tree, (a2), has leaves. The third tree, (a3), has 7 total leaves. How many total leaves will be on the 7th tree?
a) 63
b) 127
c) 128
d) 256

Answers

Is b jsjsj nsjsjaknajdudiensjkalaljs

Answer:

The total leaves on the 7th tree is, 127.

Explanation:

Definition of sequence and series:

A sequence is defined as a set of integers arranged in a specific order. A series, on the other hand, is defined as the sum of a sequence's elements.

Step by step:

The total leave will be on  7th tree are calculated.

Multiply term 2 and add term 1, to get the next digit.

According, to the question:

[tex]1[/tex]×[tex]2+1=3[/tex]

[tex]3[/tex]×[tex]2+1=7[/tex]

[tex]7[/tex]×[tex]2+1=15[/tex]

[tex]15[/tex]×[tex]2+1=31[/tex]

[tex]31[/tex]×[tex]2+1=63[/tex]

[tex]63[/tex]×[tex]2+1=127[/tex]

Hence, the 7th tree will have a total of 127 leaves.

To know more about sequence and series here, https://brainly.com/question/7882626

#SPJ2

Elizabeth bought snacks for her team's practice. She bought a bag of apples for $3.41
and a 10-pack of juice bottles. The total cost before tax was $13.21. Write and solve an
equation which can be used to determine j, how much each bottle of juice costs?

Answers

Answer:

$9.80

Step-by-step explanation:

$13.21 - $3.41 = $9.80

Answer:$5.49

Step-by-step explanation:

   

what is the lateral of a cone with 10 yd and 12.1 yd

Answers

Answer:

190.07 yd²

Step-by-step explanation:

Given that :

Base of cone = 10 yards

Height of cone = 12.1 yards

The lateral area of a cone is obtained using the relation :

π × r * l

Where r = Radius = base / 2

l = height of cone

Radius, r = 10/ 2 = 5 yards

Lateral area = π * 5 * 12.1

Lateral area = 190. 0666

Lateral area = 190.07 yd²

Other Questions
I just wanna pay my billsRappin bout the way I feelI just wanna make a couple millLeave it to the fam in the willI just wanna sign a record dealMaybe buy a house up in the hillsMight not be the best in my field But I guarantee that ima die realHonestly just wow True or false ? Only European countries were practicing imperialism in order to build their empires. (Please explain if its false) hello weebs ;) if you have watched haikyuu i want you opinion. how do you feel abaout Shratorizawa? if you like them whos your fav player. if you dont tell me why and your least fav player on that team. hello please help thanks I NEED HELP ASAP PLEASE!! I WILL AWARD BRAINLIESTNO LINKS PLEASE AND NO WEIRD ANSWERS PLEASE I HAVE A TEST TOMMAROW DO AS MANY AS YOU WANTI AWARD BRAINLIEST PLZZ HELP ME!!!!!!!!What was a challenge to U.S. forces in the Vietnam War? a shortage of soldiers lack of a defined front lack of modern weaponry a shortage of supplies 1. Explain "haploid"pairs. IM REALLY LOST ( no links!) these questions seem harder than usual im sorry for the bother 2x2 5xy 6 + 4x2 3xy + 1 x2y A carpenter attaches a piece of plywood with a width of 4 feet and a length of p to another piece of plywood that has a width of 4 feet and a length of 8 feet.Which expression represents the area of the combined piece of plywood? A. 4(p + 8) B. 8(p + 4) C. 8(p + 8) D. 4(p + 4) Write an expression that represents theperimeter of this picture frame. Work out the area of this quarter circle of radius 8 cm.Give your answer in terms of pie . The primary ethnic group of Iran is What is the probability of scoring more than 3 Which middle colony was owned by the Netherlands originally?O VirginiaO New YorkO MarylandO Pennsylvania The Albany Plan was created by the British to conquer:FranceSpainNew AmsterdamCanada Find the productGive your answer in simplest form.3/5x12 Barb purchased a loaf of bread for $2 and p pounds of sliced gam at $5 per pound for a total of 13.25. The relationship between what she purchased and her total purchase price is represented by the equation 5p + 2 = 13.25. What was the total number of pounds of ham that Barb purchased? Could Checks and balances exist together with equal application of the law